Under given conditions whether $limlimits_{nto infty}...












3












$begingroup$



Let ${f_n}_{n=1}^{infty}$ be a sequence of continuous real-valued functions defined on $mathbb R$ which converges pointwise to a continuous real-valued function $f$. Which of the following statements are true?



(i) If $0leq f_n leq f$ for all $nin mathbb N$ then $displaystyle lim_{nto infty} int_{-infty}^{infty}f_n(t)dt=int_{-infty}^{infty}f(t)dt.$



(ii) If $|f_n(t)|leq |sin t|$ for all $tin mathbb R$ and for all $nin mathbb N,$ then $displaystyle lim_{nto infty} int_{-infty}^{infty}f_n(t)dt=int_{-infty}^{infty}f(t)dt.$




(i) If $int_{-infty}^{infty} f<infty$, then we can use DOMINATED CONVERGENCE theorem and can say that the statement is true. But if $int_{-infty}^{infty}f=infty$ then what can we say about the statement?



(ii) I was not able to do this one.



Note: At the answer-key it's given that (i) is true but (ii) is false.










share|cite|improve this question











$endgroup$

















    3












    $begingroup$



    Let ${f_n}_{n=1}^{infty}$ be a sequence of continuous real-valued functions defined on $mathbb R$ which converges pointwise to a continuous real-valued function $f$. Which of the following statements are true?



    (i) If $0leq f_n leq f$ for all $nin mathbb N$ then $displaystyle lim_{nto infty} int_{-infty}^{infty}f_n(t)dt=int_{-infty}^{infty}f(t)dt.$



    (ii) If $|f_n(t)|leq |sin t|$ for all $tin mathbb R$ and for all $nin mathbb N,$ then $displaystyle lim_{nto infty} int_{-infty}^{infty}f_n(t)dt=int_{-infty}^{infty}f(t)dt.$




    (i) If $int_{-infty}^{infty} f<infty$, then we can use DOMINATED CONVERGENCE theorem and can say that the statement is true. But if $int_{-infty}^{infty}f=infty$ then what can we say about the statement?



    (ii) I was not able to do this one.



    Note: At the answer-key it's given that (i) is true but (ii) is false.










    share|cite|improve this question











    $endgroup$















      3












      3








      3


      2



      $begingroup$



      Let ${f_n}_{n=1}^{infty}$ be a sequence of continuous real-valued functions defined on $mathbb R$ which converges pointwise to a continuous real-valued function $f$. Which of the following statements are true?



      (i) If $0leq f_n leq f$ for all $nin mathbb N$ then $displaystyle lim_{nto infty} int_{-infty}^{infty}f_n(t)dt=int_{-infty}^{infty}f(t)dt.$



      (ii) If $|f_n(t)|leq |sin t|$ for all $tin mathbb R$ and for all $nin mathbb N,$ then $displaystyle lim_{nto infty} int_{-infty}^{infty}f_n(t)dt=int_{-infty}^{infty}f(t)dt.$




      (i) If $int_{-infty}^{infty} f<infty$, then we can use DOMINATED CONVERGENCE theorem and can say that the statement is true. But if $int_{-infty}^{infty}f=infty$ then what can we say about the statement?



      (ii) I was not able to do this one.



      Note: At the answer-key it's given that (i) is true but (ii) is false.










      share|cite|improve this question











      $endgroup$





      Let ${f_n}_{n=1}^{infty}$ be a sequence of continuous real-valued functions defined on $mathbb R$ which converges pointwise to a continuous real-valued function $f$. Which of the following statements are true?



      (i) If $0leq f_n leq f$ for all $nin mathbb N$ then $displaystyle lim_{nto infty} int_{-infty}^{infty}f_n(t)dt=int_{-infty}^{infty}f(t)dt.$



      (ii) If $|f_n(t)|leq |sin t|$ for all $tin mathbb R$ and for all $nin mathbb N,$ then $displaystyle lim_{nto infty} int_{-infty}^{infty}f_n(t)dt=int_{-infty}^{infty}f(t)dt.$




      (i) If $int_{-infty}^{infty} f<infty$, then we can use DOMINATED CONVERGENCE theorem and can say that the statement is true. But if $int_{-infty}^{infty}f=infty$ then what can we say about the statement?



      (ii) I was not able to do this one.



      Note: At the answer-key it's given that (i) is true but (ii) is false.







      integration sequence-of-function






      share|cite|improve this question















      share|cite|improve this question













      share|cite|improve this question




      share|cite|improve this question








      edited Jan 10 at 16:19









      Did

      248k23224463




      248k23224463










      asked Jan 10 at 16:10









      nurun neshanurun nesha

      1,0362623




      1,0362623






















          1 Answer
          1






          active

          oldest

          votes


















          2












          $begingroup$

          (1) Suppose $0 le f_n le f$ and $int_{-infty}^infty f(t); dt = infty$. Given $N > 0$, there is $M$ such that $int_{-M}^M f(t); dt > N$. By dominated convergence $int_{-M}^M f_n(t); dt to int_{-M}^M f(t); dt$, so $int_{-infty}^infty f_n(t); dt ge int_{-M}^M f_n(t); dt > N$ for sufficiently large $n$.



          (2) Try $f_n(t) = sin(t)$ for $n pi < t < (n+1)pi$, $0$ otherwise.






          share|cite|improve this answer









          $endgroup$









          • 3




            $begingroup$
            $f$ is stated to be a continuous real-valued function.
            $endgroup$
            – Robert Israel
            Jan 10 at 16:25













          Your Answer





          StackExchange.ifUsing("editor", function () {
          return StackExchange.using("mathjaxEditing", function () {
          StackExchange.MarkdownEditor.creationCallbacks.add(function (editor, postfix) {
          StackExchange.mathjaxEditing.prepareWmdForMathJax(editor, postfix, [["$", "$"], ["\\(","\\)"]]);
          });
          });
          }, "mathjax-editing");

          StackExchange.ready(function() {
          var channelOptions = {
          tags: "".split(" "),
          id: "69"
          };
          initTagRenderer("".split(" "), "".split(" "), channelOptions);

          StackExchange.using("externalEditor", function() {
          // Have to fire editor after snippets, if snippets enabled
          if (StackExchange.settings.snippets.snippetsEnabled) {
          StackExchange.using("snippets", function() {
          createEditor();
          });
          }
          else {
          createEditor();
          }
          });

          function createEditor() {
          StackExchange.prepareEditor({
          heartbeatType: 'answer',
          autoActivateHeartbeat: false,
          convertImagesToLinks: true,
          noModals: true,
          showLowRepImageUploadWarning: true,
          reputationToPostImages: 10,
          bindNavPrevention: true,
          postfix: "",
          imageUploader: {
          brandingHtml: "Powered by u003ca class="icon-imgur-white" href="https://imgur.com/"u003eu003c/au003e",
          contentPolicyHtml: "User contributions licensed under u003ca href="https://creativecommons.org/licenses/by-sa/3.0/"u003ecc by-sa 3.0 with attribution requiredu003c/au003e u003ca href="https://stackoverflow.com/legal/content-policy"u003e(content policy)u003c/au003e",
          allowUrls: true
          },
          noCode: true, onDemand: true,
          discardSelector: ".discard-answer"
          ,immediatelyShowMarkdownHelp:true
          });


          }
          });














          draft saved

          draft discarded


















          StackExchange.ready(
          function () {
          StackExchange.openid.initPostLogin('.new-post-login', 'https%3a%2f%2fmath.stackexchange.com%2fquestions%2f3068827%2funder-given-conditions-whether-lim-limits-n-to-infty-int-infty-infty%23new-answer', 'question_page');
          }
          );

          Post as a guest















          Required, but never shown

























          1 Answer
          1






          active

          oldest

          votes








          1 Answer
          1






          active

          oldest

          votes









          active

          oldest

          votes






          active

          oldest

          votes









          2












          $begingroup$

          (1) Suppose $0 le f_n le f$ and $int_{-infty}^infty f(t); dt = infty$. Given $N > 0$, there is $M$ such that $int_{-M}^M f(t); dt > N$. By dominated convergence $int_{-M}^M f_n(t); dt to int_{-M}^M f(t); dt$, so $int_{-infty}^infty f_n(t); dt ge int_{-M}^M f_n(t); dt > N$ for sufficiently large $n$.



          (2) Try $f_n(t) = sin(t)$ for $n pi < t < (n+1)pi$, $0$ otherwise.






          share|cite|improve this answer









          $endgroup$









          • 3




            $begingroup$
            $f$ is stated to be a continuous real-valued function.
            $endgroup$
            – Robert Israel
            Jan 10 at 16:25


















          2












          $begingroup$

          (1) Suppose $0 le f_n le f$ and $int_{-infty}^infty f(t); dt = infty$. Given $N > 0$, there is $M$ such that $int_{-M}^M f(t); dt > N$. By dominated convergence $int_{-M}^M f_n(t); dt to int_{-M}^M f(t); dt$, so $int_{-infty}^infty f_n(t); dt ge int_{-M}^M f_n(t); dt > N$ for sufficiently large $n$.



          (2) Try $f_n(t) = sin(t)$ for $n pi < t < (n+1)pi$, $0$ otherwise.






          share|cite|improve this answer









          $endgroup$









          • 3




            $begingroup$
            $f$ is stated to be a continuous real-valued function.
            $endgroup$
            – Robert Israel
            Jan 10 at 16:25
















          2












          2








          2





          $begingroup$

          (1) Suppose $0 le f_n le f$ and $int_{-infty}^infty f(t); dt = infty$. Given $N > 0$, there is $M$ such that $int_{-M}^M f(t); dt > N$. By dominated convergence $int_{-M}^M f_n(t); dt to int_{-M}^M f(t); dt$, so $int_{-infty}^infty f_n(t); dt ge int_{-M}^M f_n(t); dt > N$ for sufficiently large $n$.



          (2) Try $f_n(t) = sin(t)$ for $n pi < t < (n+1)pi$, $0$ otherwise.






          share|cite|improve this answer









          $endgroup$



          (1) Suppose $0 le f_n le f$ and $int_{-infty}^infty f(t); dt = infty$. Given $N > 0$, there is $M$ such that $int_{-M}^M f(t); dt > N$. By dominated convergence $int_{-M}^M f_n(t); dt to int_{-M}^M f(t); dt$, so $int_{-infty}^infty f_n(t); dt ge int_{-M}^M f_n(t); dt > N$ for sufficiently large $n$.



          (2) Try $f_n(t) = sin(t)$ for $n pi < t < (n+1)pi$, $0$ otherwise.







          share|cite|improve this answer












          share|cite|improve this answer



          share|cite|improve this answer










          answered Jan 10 at 16:20









          Robert IsraelRobert Israel

          326k23215469




          326k23215469








          • 3




            $begingroup$
            $f$ is stated to be a continuous real-valued function.
            $endgroup$
            – Robert Israel
            Jan 10 at 16:25
















          • 3




            $begingroup$
            $f$ is stated to be a continuous real-valued function.
            $endgroup$
            – Robert Israel
            Jan 10 at 16:25










          3




          3




          $begingroup$
          $f$ is stated to be a continuous real-valued function.
          $endgroup$
          – Robert Israel
          Jan 10 at 16:25






          $begingroup$
          $f$ is stated to be a continuous real-valued function.
          $endgroup$
          – Robert Israel
          Jan 10 at 16:25




















          draft saved

          draft discarded




















































          Thanks for contributing an answer to Mathematics Stack Exchange!


          • Please be sure to answer the question. Provide details and share your research!

          But avoid



          • Asking for help, clarification, or responding to other answers.

          • Making statements based on opinion; back them up with references or personal experience.


          Use MathJax to format equations. MathJax reference.


          To learn more, see our tips on writing great answers.




          draft saved


          draft discarded














          StackExchange.ready(
          function () {
          StackExchange.openid.initPostLogin('.new-post-login', 'https%3a%2f%2fmath.stackexchange.com%2fquestions%2f3068827%2funder-given-conditions-whether-lim-limits-n-to-infty-int-infty-infty%23new-answer', 'question_page');
          }
          );

          Post as a guest















          Required, but never shown





















































          Required, but never shown














          Required, but never shown












          Required, but never shown







          Required, but never shown

































          Required, but never shown














          Required, but never shown












          Required, but never shown







          Required, but never shown







          Popular posts from this blog

          Human spaceflight

          Can not write log (Is /dev/pts mounted?) - openpty in Ubuntu-on-Windows?

          File:DeusFollowingSea.jpg